Vector-Valued Function Ellipse

  • Thread starter Thread starter vandersmissen
  • Start date Start date
  • Tags Tags
    Ellipse Function
Click For Summary
The discussion focuses on finding a vector-valued function for the ellipse defined by the equation 4x² + 9y² = 36, which simplifies to x²/9 + y²/4 = 1. The correct parametric equations for tracing the ellipse counterclockwise are x = 3cos(t) and y = 2sin(t), leading to the vector form 3cos(t)i + 2sin(t)j. For a clockwise traversal, the function is adjusted to 3cos(t) - 2sin(t)j, utilizing the properties of sine and cosine. The solutions provided confirm the correctness of the approach to represent the ellipse in both directions.
vandersmissen
Messages
19
Reaction score
0

Homework Statement


Find a vector-valued function f that traces out the given curve in the indicated direction.
(a) Counterclockwise (b) Clockwise.

4x2+9y2=36


Homework Equations



x2+y2=r2
cos2t+sin2t=1

The Attempt at a Solution


From what I can determine, this is an ellipse. I believe this is how the answer is found.

4x2/36+9y2/36=36/36

x2/9+y2/4=1

so cos2t+sin2t=1 and x2/9+y2/4=1
this means x=3cos(t) and y=2sin(t) to balance it equivalently, I think.

putting that in component form, 3cos(t)i+2sin(t)j for counterclockwise and 3cos(t)i-2sin(t)j for clockwise

That does equal the correct answer , but I am not sure if I got to it correctly. Can someone please review my work and let me know if I am approaching this correctly, I do not show anything in my notes or in my book on how to perform this kind of equation.
 
Physics news on Phys.org
Yes, that is perfectly correct. Notice that from the definition of sine and cosine in terms of the unit circle, "t" goes around the circle counterclockwise so your 3cos(t)i+ 2sin(t)j does describe the circle counterclockwise as t increases. To go around the circle clockwise, use -t instead: 3cos(-t)+ 2sin(-t)= 3cos(t)- 2sin(-t) because cosine is an even function and sine is an odd function.
 
Question: A clock's minute hand has length 4 and its hour hand has length 3. What is the distance between the tips at the moment when it is increasing most rapidly?(Putnam Exam Question) Answer: Making assumption that both the hands moves at constant angular velocities, the answer is ## \sqrt{7} .## But don't you think this assumption is somewhat doubtful and wrong?

Similar threads

Replies
3
Views
2K
  • · Replies 5 ·
Replies
5
Views
3K
  • · Replies 3 ·
Replies
3
Views
2K
Replies
12
Views
2K
Replies
2
Views
2K
  • · Replies 4 ·
Replies
4
Views
2K
  • · Replies 1 ·
Replies
1
Views
1K
  • · Replies 4 ·
Replies
4
Views
2K
  • · Replies 1 ·
Replies
1
Views
2K
Replies
1
Views
9K